Fórum de Matemática
DÚVIDAS? Nós respondemos!

Um Fórum em Português dedicado à Matemática
Data/Hora: 03 jun 2024, 20:50

Os Horários são TMG [ DST ]




Fazer Nova Pergunta Responder a este Tópico  [ 4 mensagens ] 
Autor Mensagem
MensagemEnviado: 14 dez 2013, 14:07 
Offline

Registado: 09 ago 2013, 14:13
Mensagens: 47
Localização: Brasil, Castanhal-PA
Agradeceu: 0 vez(es)
Foi agradecido: 2 vezes
(Livro: Cálculo - Autor: James Stewart - Volume 2 - 7ª Edição - Q. 17 - Pág.: 925)
Utilize coordenadas cilíndricas.
Calcule \(\iiint_E \sqrt{x^2+y^2}dV\), onde E é a região que está dentro do cilindro x²+y²=16 e entre os planos z=-5 e z=4.


Resposta: \(384\pi\)

_________________
Vide ultra
_______________________________________________________________________________________

Imagem


Editado pela última vez por raimundojr em 14 dez 2013, 18:05, num total de 1 vez.

Topo
 Perfil  
 
MensagemEnviado: 14 dez 2013, 15:12 
Offline

Registado: 27 jul 2013, 15:50
Mensagens: 56
Localização: Rio de Janeiro
Agradeceu: 4 vezes
Foi agradecido: 44 vezes
Essa questão é simples, mas demanda um pouco de trabalho braçal...

Inicialmente temos que observar que essa integral se dividirá em 3, uma vez que existem 3 superfícies esféricas distintas. A saber:

\(\large i)0\leq\varphi\leq\frac{\pi}{4}: 0\leq z\leq 4\rightarrow 0\leq\rho\leq\frac{4}{\cos(\varphi)};\)
\(\large ii)\frac{\pi}{4}\leq\varphi\leq\pi-\arctan(\frac{4}{5}):0\leq\rho\leq\frac{2}{\sin(\varphi)};\)
\(\large iii)\pi-\arctan(\frac{4}{5})\leq\varphi\leq\pi:0\leq\rho\leq\frac{-5}{\cos(\varphi)};\)

O integrando: \(\large \sqrt{x^2+y^2}=\sqrt{\rho^2\sin^2(\varphi)}=\rho\sin(\varphi)\rightarrow\sqrt{x^2+y^2}dV=\rho^3\sin^2(\varphi)d\rho d\varphi d\theta\)

Daí vem que:

\(\large \iiint_{E}\sqrt{x^2+y^2}dV=\int_{0}^{2\pi}\int_{0}^{\frac{\pi}{4}}\int_{0}^{\frac{4}{\cos(\varphi)}}\rho^3\sin^2(\varphi)d\rho d\varphi d\theta+\int_{0}^{2\pi}\int_{\frac{\pi}{4}}^{\pi-\arctan(\frac{4}{5})}\int_{0}^{\frac{2}{\sin(\varphi)}}\rho^3\sin^2(\varphi)d\rho d\varphi d\theta+\int_{0}^{2\pi}\int_{\pi-\arctan(\frac{4}{5})}^{\pi}\int_{0}^{\frac{-5}{\cos(\varphi)}}\rho^3\sin^2(\varphi)d\rho d\varphi d\theta\)

Agora é conta e mais contas...

Espero ter ajudado,
Qualquer dúvida sinalize.

_________________
"A Matemática é a linguagem com o qual Deus escreveu o universo"
Galileu Galilei


Topo
 Perfil  
 
MensagemEnviado: 14 dez 2013, 18:45 
Offline

Registado: 21 jul 2013, 00:22
Mensagens: 673
Localização: Manchester
Agradeceu: 93 vezes
Foi agradecido: 340 vezes
Por coordenadas cilíndricas:

como sabemos os limites dz, vamos dy e dx, o cilindro \(x^2+y^2=16\), no plano \(xy\) determinam uma circuferência de raio 4 :

\(\int_{-4}^{4} \; \int_{-\sqrt{16-x^2}}^{\sqrt{16-x^2}}\;\int_{-5}^{4} \; \sqrt{x^2+y^2}\; dzdydx\)


passando para coordendas polares: \(x=rcos\theta \;\; y=rsen\theta \;\; \text{Jacobiano}=r\) :


\(\int_{0}^{2\pi} \; \int_{0}^{4} \;\int_{-5}^{4} r^2 \; dr d\theta\)


Topo
 Perfil  
 
MensagemEnviado: 14 dez 2013, 21:19 
Offline

Registado: 27 jul 2013, 15:50
Mensagens: 56
Localização: Rio de Janeiro
Agradeceu: 4 vezes
Foi agradecido: 44 vezes
Nossa, eu misturei tudo...
É por CILÍNDRICAS e não por esféricas....
desconsidere minha postagem então...

_________________
"A Matemática é a linguagem com o qual Deus escreveu o universo"
Galileu Galilei


Topo
 Perfil  
 
Mostrar mensagens anteriores:  Ordenar por  
Fazer Nova Pergunta Responder a este Tópico  [ 4 mensagens ] 

Os Horários são TMG [ DST ]


Quem está ligado:

Utilizadores a ver este Fórum: Nenhum utilizador registado e 6 visitantes


Criar perguntas: Proibído
Responder a perguntas: Proibído
Editar Mensagens: Proibído
Apagar Mensagens: Proibído
Enviar anexos: Proibído

Pesquisar por:
Ir para: